chaîne arithmétique

13

Réponses


  • Re: chaîne arithmétique
    Auteurs: gilles benson (---.w80-11.abo.wanadoo.fr)
    Date: 01-06-07 21:12

    Rebonsoir, comme je suis sorti en promenade avec mon labrador, je pense avoir quelques idées :
    on a:
    $\displaystyle a^n - b^n = (a - b)(\sum_{i=0}^{n-1} a^{n-i-1}b^{i})$
    avec $ n$ impair et $ b= -1$, on obtient :
    $\displaystyle a^n +1= (a +1)(\sum_{i=0}^{n-1} a^{n-i-1} (-1)^{i})$
    Avec $ p$ et $ q$ impairs, on obtient en écrivant $ a=x^p$ et $ b= {(-1)}^p$ :
    $\displaystyle {(x^p)}^q +1= ( x^p + 1)( \sum_{i=0}^{q-1} {(x^p)}^{q - i - 1}{(-1)}^{i} )$
    En prenant $ x=2$, on obtient :
    $\displaystyle {(2^p)}^q +1= ( 2^p + 1)( \sum_{i=0}^{q-1} {(2^p)}^{q - i - 1}{(-1)}^{i} )$
    donc $ 2^p +1$ divise $ 2^{pq} +1$ si $ p$ et $ q$ sont impairs.
    On peut noter aussi avec $ a=2$ :
    $\displaystyle 2^n +1= 3(\sum_{i=0}^{n-1} 2^{n-i-1}{(-1)}^{i})$
    donc si $ n$ est impair, 3 divise $ 2^n +1$

    Bilan partiel : $ 2^{pq} +1$ admet au moins trois diviseurs distincts et il me reste à comparer les tailles respectives de ces différents nombres pour en trouver un quatrième

    Re: chaîne arithmétique
    Auteurs: galax (---.adsl.proxad.net)
    Date: 01-06-07 21:43

    Bonjour Gilles,

    Tout d'abord la divisibilité par 3 de $ 2^a + 1 $ pour un a impair peut être montré plus rapidement :
    $ 2^a = (-1)^a =-1 \pmod 3$
    donc on a bien que $ 2^a + 1 =0 \pmod 3$.

    Ainsi pour tout premier $p>3, 2^p + 1 $ a au mois 4 diviseurs.
    A savoir $1, 3, \frac {2^p + 1}{3}, 2^p + 1 $.

    Remarque : pour $p=3,\ 2^p + 1 = 9 $ on n'a que trois diviseurs : $1, 3 , 9$
    Comme me l'a fait remarquer Borde dans son message du 12-29-06 12:17, et où il m'a rappelé également la terminologie en usage en France.

    Bonne continuation.
    Galax

    Re: chaîne arithmétique
    Auteurs: gilles benson (---.w193-253.abo.wanadoo.fr)
    Date: 01-07-07 00:25

    Bonsoir Galax, effectivement employer les congruences est sûrement une bonne idée en arithmétique... ceci dit, je n'y peux rien, l'identité géométrique sous toutes ses formes me fascine.
    Quand j'ai écrit:
    si $ n$ est impair, $3 \mid 2^n +1$
    Cela donnait évidemment 3 diviseurs pour $ 2^3 + 1$ mais 4 au moins pour tout entier impair supérieur à 3 puisque l'on a deux diviseurs propres...

    Entre temps, j'ai vérifié que:
    $\displaystyle 2^{ab}+1 > (2^a+1)(2^b+1)$
    si $ a$ et $ b$ sont deux entiers supérieurs ou égaux à 5, ce qui entraine le fait que $ 2^{pq} +1$ admet au moins 4 diviseurs propres, soit $ \displaystyle d(2^{pq} +1) \geq 2^4 = 4^2$ en notant $ d(n)$ le nombre de diviseurs de l'entier $ n$ (toujours avec $ p$ et $ q$ des nombres premiers distincts supérieurs ou égaux à 5).

    Bon, il fera quand même jour demain mais je suis loin du compte.

    Re: chaîne arithmétique
    Auteurs: galax (---.adsl.proxad.net)
    Date: 01-07-07 01:15

    Bonsoir Gilles,

    Pour la suite de la démonstration on pourra utiliser une récurrence.

    Sincèrement,
    Galax
  • ok, merci Galax, je vais essayer d'y songer.
    A demon  wind propelled me east of the sun
  • Bonjour Gilles,

    A propos du problème 14

    J'ai recopié les derniers postes d'hier (avec peu de succès) pour pouvoir te répondre.

    Pour poursuivre les raisonnements de ton poste du 01-07-07 00:25, tu peux utiliser les 2 lemmes suivants:
    lemme 3:
    Si $ 3 \nmid n$, alors $ 9 \nmid 2^n +1$ ( pour en n impair )
    lemme 4:
    Si pgcd(m,n) = 1, avec m en n impaires, alors pgdc($2^m +1$,$2^n +1$) =3.

    N.B. Je considère que les deux raisonnements précédant, déjà démontrés, peuvent être considérés comme lelemme 1 et le lemme 2.

    lemme 1:
    $ 3 | 2^n + 1 $ pour un n impair
    lemme 2: Si pgdc(m,n)
    Pour tout premier p>3, $ 2^p + 1$ a au mois 4 diviseurs.

    Sincèrement,

    Galax

    *********

    bonjour AD,

    j'ai quelques problèmes avec les messages en latex.
    Merci d'avance pour ton aide.

    Sincèrement,

    Galax
  • ok merci encore; avec ces résultats, les choses s'éclaircissent notablement; ceci dit je vais être très occupé durant les trois jours qui suivent; d'un autre côté la question du pgcd me taraudait un peu mais les attaques que j'avais envisagées étaient vraiment faibles.
    A demon  wind propelled me east of the sun
  • gilles,

    tes idées sont très valables, j'ai mis ces 4 lemmes pour mettre au clair le travail que tu a déjà ( presque) fait.

    Sincèrement,

    Galax
  • bonjour, pour le lemme 3:
    Si $ 3 \nmid n$, alors $ 9 \nmid 2^n +1$ ( pour en n impair )

    j'ai eu le temps de réfléchir(!):

    si $n$ est impair et que $ 3 \nmid n$, $n$ est de la forme $\displaystyle 6k+1 \text{ ou } 6k+5$; donc $2^n+1$ est de la forme:
    $\displaystyle 2 \times {64}^k+1 \text{ ou } 32 \times {64}^k+1$.

    Puisque 64 est congru à 1 modulo 9 (je ne sais pas taper modulo en latex...), le résultat en découle.
    A demon  wind propelled me east of the sun
  • bonjour gilles,

    Ton idée pour le lemme 3 est très bien.

    Personnelement j'ai fait presque la même démonstration que toi.
    J'ai pris n de la forme $ \displaystyle 3k+1$ ou $ \displaystyle 3k+2$

    Sincèrement,

    Galax
  • Bonsoir Galax, en obligeant mon labrador à effectuer sa promenade prévespérale, j'ai obtenu la solution du lemme 4:
    Si $\mathrm{pgcd}(m,n) = 1$, avec $m, n$ entiers impairs, alors $\mathrm{pgdc}( 2^m +1,2^n +1) = 3$.
    On peut écrire $ \displaystyle m > n \text{ et } m=an+r \text{ avec } 0 \leq r < n$.
    Par suite :
    $$ 2^m+1 - 2^n+1 = 2^n(2^{m-n}-1)$$
    Avec les conditions sur $n \text{ et } m$, on a :
    un nombre premier (impair) $q \mid 2^m+1 \text{ et } 2^n+1$ entraine :
    $$q\mid 2^{m-n}-1$$ Si $a=1$, on a obtenu :
    $$q\mid 2^m+1\text{ et } 2^r-1$$ Sinon :
    $$q\mid 2^{m-n}-1 \text{ et } 2^n+1 \text{ entraine } q\mid 2^n(2^{m-2n}+1)\text{ par addition }$$ Donc :
    $$ q\mid 2^{m-2n}+1$$ On voit donc que l'on a prouvé :
    $$q\mid 2^r-1 \text{ ou } 2^r+1 $$
    Par suite, en continuant à additionner ou soustraire, on arrive via l'algorithme d'Euclide à :
    $$q \mid 2^{\mathrm{pgcd} (m,n)}-1 \text{ ou } 2^{ \mathrm{pgcd} (m,n)}+1 $$
    On a $\mathrm{pgcd}(m,n)=1$ ce qui entraine en fait :
    $$q\mid 1 \text{ ou } q \mid 3 $$
    Il me reste à faire la synthèse de tout cela.
    A demon  wind propelled me east of the sun
  • bonjour gilles,

    Oui tes idées sont très bien,

    Moi après beaucoup des relectures de ma solution je suis arrivé à quelque chose plus court:

    tout d'abord d'après le lemme 1

    3|pgdc($ 2^m +1$,$ 2^n +1$)

    Je pose D=pgdc($ 2^m +1$,$ 2^n +1$).

    Comme $ (2^m +1)(2^m -1)=2^{2m} -1$ et $ (2^n +1)(2^n -1)=2^{2n} -1$,

    on a D|pgdc($2^{2m} -1, 2^{2m} -1$)= $2^{pgdc(2m, 2n)}$ - 1 = $2^2$ - 1 = 3

    donc D=3.

    N.B. J'utilise le fait bien connu :
    pgdc($2^{2m} -1, 2^{2m} -1$)= $2^{pgdc(2m, 2n)}- 1$


    Maintenant, il reste à prouver le problème 14 lui même.

    Sincèrement,

    Galax
  • finalement, et je l'admets après avoir tourné ce truc dans tous les sens:
    j'ai vérifié antérieurement que:

    $\displaystyle 2^{ab}+1 > (2^a+1)(2^b+1)$

    si $ a$ et $ b$ sont deux entiers supérieurs ou égaux à 5, ce qui prouve que dans le cas où $a$ et $b$ sont impairs, supérieurs à 3 et premiers entre eux, $\displaystyle 2^{ab}+1 $ admet un diviseur $q$ tel que:

    $$ 2^{ab}+1 =q(2^a+1)(2^b+1)$$

    On doit pouvoir finaliser une récurrence sur $n$ si $\displaystyle q_n=2^{\prod_{k=1}^{n}p_i}+1$:

    posons $\displaystyle a=\prod_{k=1}^{n}p_i$ et $b=p_{n+1}$ où les $p_i$ sont premiers et supérieurs à 3.
    L'hypothèse est que $2^a+1$ admet au moins $4^n$ diviseurs; on obtient donc des listes de $4^n$ diviseurs de $\displaystyle 2^{ab}+1$ en multipliant ces diviseurs
    par:
    1) 1
    2) $2^b+1$
    3) $\frac{2^b+1}{3}$
    4) $q$
    puisque $q$ est différent de 3.
    Cela donne $4^{n+1}$ diviseurs au moins pour $\displaystyle q_{n+1}=2^{\prod_{k=1}^{n+1}p_i}+1$:
    A demon  wind propelled me east of the sun
  • bonsoir Gilles,
    Maintenant c'est à ton tour de nous poser un nouveau problème. :)
    Sinon dans ta démonstration il y a encore un détail à régler.

    Bonsoir Borde >>>>> si tu as une autre approche pour ce problème 14 tu pourras en dire un mot. :)

    Sincèrement,
    Galax
  • bonsoir Galax, pourrais-tu être un peu plus explicite: est-ce un détail ou bien le fondement même de mon édifice est-il à revoir? En fait, mon principal problème était le statut du diviseur $q$ par rapport aux diviseurs de $ \displaystyle q_n=2^{\prod_{k=1}^{n}p_i}+1$ mais il me semble que j'avais vu de fantômes le concernant.
    bonne soirée en tout cas, les bras de Morphée m'appellent.
    A demon  wind propelled me east of the sun
  • bonsoir gilles,

    suite à ton dernier message je me permet donner une version de démonstration s'inspirant de la tienne.( je ne cite pas ici l'endroit où on utilise les lemmes)

    Tu as vérifié antérieurement que:

    $ \displaystyle 2^{ab}+1 > (2^a+1)(2^b+1)$

    si $ a$ et $ b$ sont deux entiers supérieurs ou égaux à 5, ce qui prouve que dans le cas où $ a$ et $ b$ sont impairs, supérieurs à 3 et premiers entre eux, $ \displaystyle 2^{ab}+1 $ admet un diviseur $ q$ tel que:

    $\displaystyle 2^{ab}+1 =q(2^a+1)\frac{2^b+1}{3}$



    On doit pouvoir finaliser une récurrence sur $ n$ si $ \displaystyle q_n=2^{\prod_{k=1}^{n}p_i}+1$:
    Tout d’abord,(on a déjà vu que ) pour tout p un nombre premier strictement supérieur à 3
    $2^p+1$ admet au mois 4 diviseurs : 1, 3, $\frac{2^p+1}{3}$, $2^p+1$

    posons $ \displaystyle a=\prod_{k=1}^{n}p_i$ et $ b=p_{n+1}$ où les $ p_i$ sont premiers et supérieurs à 3.

    L'hypothèse est que $ 2^a+1$ admet au moins $ 4^n$ diviseurs; on obtient donc des listes de $ 4^n$ diviseurs de $ \displaystyle 2^{ab}+1 $ en multipliant ces diviseurs
    par:
    1) 1
    2) $ \frac{2^b+1}{3}$
    3) $ q$ ( on note que q est différent de 3 )
    4) $q \frac{2^b+1}{3}$

    Cela donne $ 4^{n+1}$ diviseurs au moins pour
    $ \displaystyle q_{n+1}=2^{\prod_{k=1}^{n+1}p_i}+1$

    (NB dans la démonstration que j'ai fait pour moi, les 4 diviseurs ont été obtenus un peu différement)
    Sincèrement,

    Galax
  • bonjour Galax, effectivement, il y avait un problème de 3 superfétatoire dans ma sélection des diviseurs et le choix que tu fournis résout cette difficulté.
    A demon  wind propelled me east of the sun
  • Bonsoir, j'ai donc le droit de poser un problème 15 avec mes 3/4 de solution du problème 14; j'ai trouvé ceci :

    On note $a(k,n)$ le nombre d'apparition du chiffre $ k $ comme premier chiffre dans l'écriture en base 10 (pas 15, pitié) de $2^p$ si $0 \leq p \leq n$.
    Ainsi $a(1,10)= 3$ à cause de 16, 128, 1024.
    Je vous demande de déterminer la limite quand $n$ tend vers l'infini de :
    $$\frac{a(k,n)}{n}$$
    Pour que Borde ne s'ennuie pas, j'ai trouvé dans un livre récent : problème 15bis :
    Prouver que si $a$ est un entier non nul et $f(x,y)$ un polynôme irréductible homogène de degré $n \geq 3$ à coefficients entiers qui n'est pas une puissance $n$-ième d'une forme linéaire ou bien de la racine carrée d'une forme quadratique, alors l'équation $$f(x,y)=0$$ admet un nombre fini de solutions.
    A demon  wind propelled me east of the sun
  • bonjour, sinon tiré d'un grand classique:

    15ter:

    on considère un entier $n$ impair au moins égal à 3; prouver que $n$ et $n+2$ sont premiers ssi $(n-1)!$ n'est divisible ni par $n$ ni par $n+2$.

    bonne journée
    A demon  wind propelled me east of the sun
  • Bonjour gilles,

    A propos du problème 15
    {\bf On note $ a(k,n)$ le nombre d'apparition du chiffre $ k $ comme premier chiffre dans l'écriture en base 10 (pas 15, pitié) de $ 2^p$ si $ 0 \leq p \leq n$.}
    Ainsi $ a(1,10)= 3$ à cause de 16, 128, 1024.
    Je vous demande de déterminer la limite quand $ n$ tend vers l'infini de : $\displaystyle \frac{a(k,n)}{n}$
    Ce problème admet une solution à l'aide de travaux de {\bf Weyl} (début du 20ième siècle )
    Mais cette solution utilise $\R/\Z$ quotient du groupe additif $\R$ par son sous-groupe $\Z$ et un argument de densité de l'image de la suite étudiée, pour aboutir aussitôt au résultat
    $$\frac{a(1,n)}{n} \ \xrightarrow[\ n \to \infty \ ]{} \ \frac{\log 2}{\log 10}$$
    Cependant personnellement je ne connais aucune démonstration purement arithmétique de ce résultat. Tu as pensé sans doute à une démonstration de type arithmétique: peux-tu nous en donner une indication ?

    Merci
    Sincèrement,

    Galax
  • Euh, c'est bien joli tout cela, mais si on revenait au problème initial concernant le nombre premier p, parce que même à coup de belles démos vous vous égarez.
  • bonsoir Galax, je vais donner la limite attendue:

    $$\lim_{n \rightarrow \infty} \frac{a(k,n)}{n}=\frac{\log(k+1)-\log(k)}{\log(10)}$$

    De plus, mieux vaut travailler sur a(k,n-1) que sur a(k,n) et on peut partir de:

    $p_n$ défini par:

    $$10^{p_n} \leq 2^n < 10^{p_n+1}$$

    et la preuve que j'ai sous les yeux utilise la notion d'équirépartition modulo 1 mais nul n'est besoin d'aller jusqu'au critère de Weyl.
    A demon  wind propelled me east of the sun
  • Bonjour gilles,

    Dans mon message précedant je n'ai donné la reponse que si k=1.(faute de frappe que je viens corriger )

    D'autre part je n'ai pas parlé de critère de Weyl mais de travaux de Weyl.

    Sincèrement,

    Galax
  • bonsoir, tu as donc la bonne solution qui mérite à mon avis d'être écrite et je suis allé jusqu'à évoquer le critère de Weyl uniquement du fait de cette faute de frappe, ceci dit je te fais toute confiance (j'ai extrait cette question de {\it Propriétés statistiques des suites arithmétiques} de Gérard Rauzy (PUF) (1976)).
    A demon  wind propelled me east of the sun
  • bonsoir gilles,

    oui, je peux rédiger en détail cette solution, mais j'ai cru qu' une solution plus arithmétique existe. (C'est aussi la raison que je n'ai pas écrit plutôt à ce sujet)

    Sincèrement,

    Galax
  • rebonsoir Galax, je donnerai la solution si quelqu'un est intéressé mais rien ne t'empêche de le faire si tu le souhaite.

    le 15 ter est soluble dans l'arithmétique élémentaire mais peut-être en connais-tu la provenance auquel cas,il ne présente que peu d'intérêt; je pensais que cette histoire de premiers jumeaux pouvait attirer l'attention de ceux qui n'étaient pas familier avec cette question et sinon, j'en suis désolé.
    Bonne soirée.
    A demon  wind propelled me east of the sun
  • Bonjour Gilles,

    Pour le 15 ter, ressemble étrangement à:

    $(n,n+2)$ sont premiers jumeaux (avec $n \geqslant 2$) $\Longleftrightarrow$ $4 \left \{ (n-1)! + 1 \right \} + n \equiv 0 \pmod {n(n+2)}$.

    référence, par exemple De Koninck-Mercier , "{\it 1001 problèmes en théorie classique des nombres}", Ellipses (2004), exercice 164 page 30.
    ...à partir du théorème de Wilson.

    Bonne journée.
  • bonsoir bs, pour le coup ma référence n'est pas celle-ci (la-dite référence traine dans ma bibliothèque mais j'avoue à ma grande honte que je ne l'ai guère ouverte.
    Il est donc tout à fait loisible d'utiliser cette équivalence qui fonctionne effectivement grace au théorème de Wilson; ceci étant une étude directe est parfaitement possible surtout pour une des deux implications; et donc pour cette question la chasse à la référence ne me paraît pas ouverte; en fait, il s'agit d'un bouquin vraiment difficile à trouver:

    W. Sierpinski: {\it 250 problèmes de théorie élémentaire des nombres} chez Hachette que j'ai eu la chance de trouver chez J. Gibert (chez qui j'ai dégotté quelques bouquins remarquables comme les trois tomes reliés de {\it Topics in complex function theory} de C.L. Siegel ou bien {\it Conformal invariants: topics in geometric function theory} de L.V. Ahlfors) voici quelques années; j'indique la côte de l'exercice: 4/118 ce qui vous ôte le souci de cette recherche.

    Remarque: le "problème 15bis peut être oublié et considéré comme une boutade pas forcément de bon aloi (avec mes excuses en fait:-( ).
    Bon courage.
    A demon  wind propelled me east of the sun
  • Bonsoir Gilles, je possède également le livre W. Sierpinski: {\it 250 problèmes de théorie élémentaire des nombres} ,mais en anglais; et je ne sais plus trop où l'avoir acheté !

    Merci pour la référence 4/118; je n'avais pas fait cet exercice, et ne vois pas trop le rapport avec ta question. Je fais cet exercice, puis, certainement que j'aurai besoin de tes éclaircissements pour comprendre le lien entre les deux exercices !

    Bonne soirée.
  • rebonsoir bs, je n'avais aucune question: La référence que j'ai donnée est celle de l'exercice que j'ai posé; en ce qui concerne le lien entre l'équivalence proposée par De Koninck-Mercier et celle de Sierpinski, cela ressemble à une coïncidence à moins de poser la question à P. Ribenboim qui est cité en référence de l'exercice de
    De Koninck-Mercier.
    Pour le coup, je vais regarder sur l'internet si je peux trouver l'édition en anglais (à supposer qu'elle soit reliée d'ailleurs).
    A demon  wind propelled me east of the sun
  • Il me semble qu'il y ait quiproquo: la solution de l'exercice 4.118 du Sierpinski n'est pas celle de ton exercice 15-ter ?
    Bonne nuit.
  • cela ressemble fort à un quiproquo en effet; cela doit tenir à mon habitude des parenthèses ouvrantes (des digressions...) imbriquées dans lesquelles je me perds moi-même. A ce sujet, le livre :{\it le manuscrit trouvé à saragosse} de Potocki est un exemple sidérant d'une telle construction littéraire (mais c'est une nouvelle digression..)En ce qui concerne le bouquin en question de Sierpinski, on peut le trouver actuellement à 189 €...Pour ce prix-là je vais faire relier mon exemplaire broché.
    Bonne soirée.
    A demon  wind propelled me east of the sun
  • bonsoir,

    Pour le problème 15, il a plus haut des indications sur la méthode et si quelqu'un veut une démonstration complète, je laisse à Gilles ( qui a proposé ce problème ) de la faire.
    ******************



    Je vous propose le

    PROBLEME 16

    des questions à propos de $\frac{1}{x} - \frac{1}{y} = \frac{m}{n} (E_m) $

    a) Montrer que $ (E_1) $ a une seule solution dans N* que si n est un nombre premier
    b) Résoudre $ (E_1) $ dans N* lorsque n=6
    c) Montrer qu'il existe un n dans N*, tel que $ (E_1) $ admet 2007 solutions exactement
    d) Trouver tous les n de N* pour lesquels $ (E_2) $ a une solution dans N* exactement.



    Sincèrement,

    Galax
  • bonjour: on avait:
    15ter:

    on considère un entier $ n$ impair au moins égal à 3; prouver que $ n$ et $ n+2$ sont premiers ssi $ (n-1)!$ n'est divisible ni par $ n$ ni par $ n+2$.

    Conscient que la référence indiquée pour ce problème n'est pas si accessible que cela, je vais rédiger une solution de cette question:

    On peut noter que tout nombre premier $p$ diviseur de $k!$ vérifie $p \leq k$ donc
    on a de manière simple:

    $n$ et $n+2$ premier $\Rightarrow$ ils ne divisent pas $(n-1)!$

    Dans l'autre sens:

    supposons donc que $n$ et $n+2$ ne divisent pas $(n-1)!$:
    puisque 3 et 5 ainsi que 5 et 7 sont jumeaux, on peut partir de $n \geq 7$ car 9 n'est pas premier.
    Si on a $n=uv$ alors $u$ et $v$ sont des diviseurs de $(n-1)!$ qui figurent tous les deux dans la liste des entiers $k$ tels que $1\leq k \leq n-1$ car $u|n \Rightarrow u < n$ si $u$ est un diviseur propre de $n$.
    On a deux cas à envisager: $u=v$ et $u \neq v$ qui mènent tous les deux à une contradiction: si $u=v$, $v^2=n >2v$ donc $2v^2|(n-1)!$ et de même sinon.

    Prouvons de la même manière que $n+2$ est premier: dans le cas contraire:

    $$n+2=uv$$ et les facteurs $u$ et $v$ sont impairs supérieurs ou égaux à 3 donc on peut écrire:
    $$3u \leq n+2 \Rightarrow u \leq \frac{n+2}{3} \leq \frac{n-1}{2} $$
    donc:
    $$2u \leq n-1$$
    et de même pour $2v$. Et on retrouve le même principe de raisonnement pour obtenir
    une contradiction sur le caractère composé de $n+2$.
    A demon  wind propelled me east of the sun
  • Finalement pour le 15:
    On note $ a(k,n)$ le nombre d'apparition du chiffre $ k $ comme premier chiffre dans l'écriture en base 10 (pas 15, pitié) de $ 2^p$ si $ 0 \leq p \leq n$.
    Ainsi $ a(1,10)= 3$ à cause de 16, 128, 1024.
    Je vous demande de déterminer la limite quand $ n$ tend vers l'infini de :

    $\displaystyle \frac{a(k,n)}{n}$

    Il mieux vaut travailler sur $a(k,n-1) = b(k,n)$ que sur $a(k,n)$ et on peut partir de:

    $ p_n$ défini par:

    $\displaystyle 10^{p_n} \leq 2^n < 10^{p_n+1}$

    Ensuite:
    $b(k,n)$ est le nombre d'entiers $q$ tels que:

    $$0 \leq q < n \text{ et } k\times {10}^{p(q)} \leq 2^q < (k+1){10}^{p(q)}$$
    soit:

    $$ \frac{\log(k)}{\log10} \leq q\frac{\log2}{\log10}- p(q) < \frac{\log(k+1)}{\log10}$$

    En remarquant que $p(q)$ est le plus grand entier inférieur ou égal à $\displaystyle \frac{\log2}{\log10}$,
    On doit voir que modulo 1, on compte le nombre de fois que le réel $q\alpha$ avec $\alpha = \frac{\log2}{\log10}$, est contenu dans l’intervalle $[ \frac{\log(k)}{\log10} ; \frac{\log(k+1)}{\log10} ]$

    Par suite, $\displaystyle \frac{b(k,n)}{n}$ est la fréquence d’apparition de ce réel $q\alpha$ (modulo 1) dans l’intervalle précité ; il reste alors à utiliser un théorème sur l’équirépartition des suites $nr$ avec $r$ irrationnel modulo 1 qui affirme que cette fréquence tend vers la longueur de l’intervalle en question ; il reste donc à prouver que $\alpha = \frac{\log2}{\log10}$ est irrationnel., ce qui se traduirait par une égalité du type $2^n=10^m$ avec $n$ et $m$ entiers naturels qui contredit le théorème fondamental de l’arithmétique.
    Ceci dit vu la fréquence avec laquelle cette question de l’équirépartition modulo 1 revient, il serait peut-être bon de la développer.
    A demon  wind propelled me east of the sun
  • bonsoir,

    Voici une indication pour le
    PROBLEME 16 (dont je rappelle ici l'énoncé )

    des questions à propos de $ \frac{1}{x} - \frac{1}{y} = \frac{m}{n} (E_m) $

    a) Montrer que $ (E_1) $ a une seule solution dans $N^{*2}$ que si n est un nombre premier
    b) Résoudre $ (E_1) $ dans $N^{*2}$ lorsque n=6
    c) Montrer qu'il existe un n dans $N^*$, tel que $ (E_1) $ admet 2007 solutions exactement dans $N^{*2}$
    d) Trouver tous les n de $N^*$ pour lesquels $ (E_2) $ a exactement une solution dans $N^{*2} $.


    Indication pour 16a):
    Montrer d'abord que pgcd(x,n-x)=1, si n est un nombre premier.

    Sincèrement,

    Galax
  • Bonjour,

    Voici une solution du PROBLEME 16a)


    Comme y doit être positif, $(E_1)$ donne:

    $\frac{1}{x} > \frac{1}{n} $

    et donc $x < n$.

    Ainsi $(E_1)$ équivaut à $y =\frac{nx}{n-x}$.

    Pour n un nombre premier on a pgcd(x,n-x) = pgcd(x,n) = 1, car $x < n$.

    D'où $\frac{nx}{n-x}$ n'est entier que si $n - x = 1$ ou encore $x = n - 1$.

    On en déduit que $x = n - 1$ et $y = n(n - 1)$ est la seule solution de l'équation

    $y =\frac{nx}{n-x}$.


    Si n n'est pas premier, n = a.b ,

    alors $x = n - a , y = b(n - a)$ est une autre solution de $(E_1)$.

    J'espère que pour la suite il y a plein d'amateurs ...

    Sincèrement,

    Galax
  • Bonjour,

    Voici les réponses pour les autres questions du problème 16:

    16b) (5, 30), (4,12), (3, 6), (2, 3) sont des solutions de $ (E_1)$ dans $ N^{*2}$ lorsque n=6
    N.B. dans $Z^{*2}$ cette équation a en plus des solutions suivantes:
    (7, - 42), (8, -24), (9, -18), (10, -15),(12, -12), (-3, -2), (-6, -3), (-12,-4), (-30, -5)

    16c) par exemple $3^{2007}$ convient

    16d) n doit être un nombre premier sauf 2, ou bien double d'un nombre premier.

    Sincèrement,

    Galax
  • Bonsoir,

    Voici un nouveau problème pour les amateurs des fonctions arithmétiques :

    {\bf PROBLEME 17

    Soit $ A_n = \sum\limits_{\substack{k=1\\ \mathrm{pgcd}(k, n)=1}}^n \dfrac{1}{k + n}$, pour tout $n \geq 1$.
    Montrer que :}
    $$A_n = \dfrac{1}{n} \sum\limits_{d\mid n}\ \sum\limits_{k=1}^d \dfrac{d}{k + d}\ \mu \left(\dfrac{n}{d}\right)$$

    Sincèrement,
    Galax
  • Indication : Soit $n \geqslant 1$ entier fixé. Utiliser la fonction indicatrice des entiers $k \in [1,n]$ tels que $\mbox {pgcd} (k,n) = 1$ qui est donnée par : $$\sum_{d \mid n, \, d \mid k} \mu(d)$$ ou bien voir mon calcul sur le fil nommé "Césaro en arithmétique".

    Borde.
  • bonjour,

    Voici le lien vers le fil nommé "Césaro en arithmétique" dont parle Borde dans son message précédant:


    http://www.les-mathematiques.net/phorum/read.php?11,357241,357651#msg-357651

    Sincèrement,

    Galax
  • Salut,
    et pourquoi ne pas simplement utiliser la formule d'inversion de Möbius~? On a~:
    \begin{eqnarray*}
    B_n&=&\sum\limits_{k=1}^n\frac{1}{k+n}\\
    &=& \sum\limits_{d\mid n}\sum\limits_{\substack{k=1\\(k,n)=d}}^n\frac{1}{k+n}\\
    &=&\sum\limits_{d\mid n}\sum\limits_{\substack{k=1\\(k,n/d)=1}}^{n/d}\frac{1}{d(k+n/d)}\\
    &=&\frac{1}{n}\sum\limits_{d\mid n}dA_d\\
    \mathrm{donc}~:\\
    nA_n&=&\sum\limits_{d\mid n}dB_d\mu(n/d)
    \end{eqnarray*}
    comme demandé.
    Watercat
  • C'est la même chose : $$A_n = \sum_{d \mid n} \mu(d) \sum_{h \leqslant n/d} \frac {1}{hd+n} = \sum_{d \mid n} \mu(n/d) \sum_{h \leqslant d} \frac {1}{nh/d+n},$$ ce qui est le résultat souhaité.

    Borde.
  • Bonjour Borde, Watercat,

    Ma solution du problème est la même que celle de Borde.

    Sinon, c'est à vous deux à proposer d'autres problèmes.

    Sincèrement,

    Galax
  • Honneur à Watercat...

    Borde.
  • Bonsoir,
    merci à Borde de me laisser la main. Voici ma question~: à quelle condition sur l'entier $n$ existe-t-il un anneau commutatif $A$ tel que le groupe des unités $A^{\times}$ soit cyclique d'ordre $n$ ?
    Watercat
  • Alooors, je propose $n$ quelconque, avec $A=\mathbb{Z}[\zeta_n]$.


    Soit $a\in A$, et soit $\rho=\max \vert a'\vert,$ où $a'$ parcourt les conjugués de $A$. La norme de $a$ est le produit de ses conjugués, et comme $a$ est un entier algébrique, sa valeur absolue est au moins $1$.

    Ainsi, $\rho\geq 1$. Un théorème de Robinson montre que si $a$ n'est pas une racine de l'unité, alors $\rho\geq \sqrt{2}>1$. La démo. n'est pas très dure, mais je n'ai pas envie de la recopier. On peut la trouver sur numdam.org dans l'article

    Cassels, John William Scott
    Sommes de racines de l'unité. Séminaire Delange-Pisot-Poitou. Théorie des nombres, 9 no. 2 (1967-1968), Exposé No. 23, 16 p.

    (Théorème 3.1)


    Ainsi, $a$ est une racine de l'unité. Il reste à voir que les seules racines de l'unité dans $A$ sont les puissances de $\zeta_n$.
    Ce qui ne doit pas être très difficile (si ce n'est pas faux, bien sûr lol)
  • Grek> Dirichlet implique que l'anneau des entiers d'un corps de nombres admet un nombre fini d'unites ssi le corps est quadratique imaginaire ou rationnel

    Joaopa
  • Salut,
    en effet, il y a une infinit\'e d'unit\'es dans $\mathbb{Z}[\zeta_n]$ d\`es que $n$ est sup\'erieur a $5$. Pour $n=5$ justement, voici ce que je propose. Supposons qu'il existe un anneau commutatif $A$ dont le groupe des unit\'es est cyclique d'ordre $5$. D'abord $A$ doit \^etre de caract\'eristique $2$, sinon $-1$ est une unit\'e d'ordre $2$ ce qui est contradictoire. Ensuite $A$ contient un \'el\'ement $u\neq 1$ tel que $u^5=1$, donc il existe un unique homomorphisme d'anneaux de $B=\mathbb{F}_{2}[X]/(X^5-1)$ dans $A$ envoyant $X$ sur $u$. La d\'ecomposition de $X^5-1$ en produit de facteurs irr\'eductibles sur $\mathbb{F}_{2}$ est $(X-1)\Phi_5(X)$ avec $\Phi_5=X^4+X^3+X^2+X+1$ (qui est irr\'eductible parce que $2$ engendre $(\mathbb{Z}/5\mathbb{Z})^{\times}$), donc :
    \begin{equation*}
    B\simeq\mathbb{F}_{2}\times\mathbb{F}_{2}[X]/(\Phi_5)
    \simeq\mathbb{F}_{2}\times\mathbb{F}_{16}.
    \end{equation*}
    Ainsi le sous-anneau de $A$ engendr\'e par $u$ doit \^etre un quotient de $B$, de sorte que son groupe des unit\'es doit \^etre ou bien trivial, ou bien cyclique d'ordre $15$. Contradiction.
    Watercat
  • Oui, je viens de m'en rendre compte il y a 30 secondes, et je revenais sur le forum pour modifier mon message. Grillé.

    Soit donc $A$ un anneau dont le groupe des unités est cyclique d'ordre $n$.
    Soit $e$ un générateur de ce groupe. Si $A$ est de caractéristique nulle, alors $\mathbb{Z}[e]$ est un sous-anneau de $A$, isomorphe à un quotient de $\mathbb{Z}[X]/(X^n -1)$, lui même isomorphe au produit des $\mathbb{Z}[\zeta_d],d\vert n$. Je ne sais pas à quoi cela pourrait servir.

    Supposons que $A$ soit de caractéristique $c>0$. Donc $\mathbb{Z}/c\mathbb{Z}$
    est un sous anneau de $A$.
    Donc $A^\times$ contient $(\mathhb{Z}/c\mathbb{Z})^\times$.


    Si $c=2^m p_1^{r_1}...p_k^{r_k},p_i$ premier impair, alors $A^\times$ contient un sous-groupe isomorphe à $$C_{2^{m-1}}\times C_{p_1^{r_1-1}(p_1-1)}\times..\times C_{p_k^{r_k-1}(p_k-1)}$$

    Remarquons que si $k\geq 2$, alors ce groupe contient un sous-groupe isomorphe au groupe de Klein, donc n'est pas cyclique. Ainsi, $k=0,1$. Si $m\geq 3$, ce groupe n'est pas cyclique non plus.

    Si $m=2$, on a n\'{e}cessairement $k=0$, sinon on a encore un sous-groupe de Klein.

    On a donc $c:=car(A)=2,p^r, 2p^r,4, p$ premier impair.
    Dans tous les cas, $\mathbb{Z}/c\mathbb{Z}[e]$ est un quotient de $\mathbb{Z}/c\mathbb{Z}[X]/(X^n-1)$

    Il faudrait donc à étudier les unités de l'anneau $\mathbb{Z}/c\mathbb{Z}[X]/(X^n-1)$ (beuh!) , ou trouver une autre astuce.


    En tout cas, en prenant $A=\mathbb{Z}/c\mathbb{Z}$, cela montre que l'on peut avoir $n=p^{r-1}(p-1)$.

    Remarquons aussi que l'on peut avoir $n=p^r-1$ en prenant le corps fini à $p^r$ éléments.


    Suite au prochain épisode..;là, je vais soulever des poids!
  • De retour de la gym...


    Remarquons que si $c=2,p^r,2p^r,4$, alors $n$ est un multiple de
    $1,p^{r-1}(p-1),p^{r-1}(p-1),2$ respectivement.

    En particulier, si $c\neq 2$, alors $n$ est forc\'{e}ment pair.
  • Bon y a un truc zarbi qui se passe avec LaTeX, je recommence...


    Soit donc $A$ un anneau dont le groupe des unités est cyclique d'ordre $n$.
    Soit $e$ un générateur de ce groupe. Si $A$ est de caractéristique nulle, alors $\mathbb{Z}[e]$ est un sous-anneau de $A$, isomorphe à un quotient de $\mathbb{Z}[X]/(X^n -1)$, lui même isomorphe au produit des $\mathbb{Z}[\zeta_d],d\vert n$. Je ne sais pas à quoi cela pourrait servir.

    Supposons que $A$ soit de caractéristique $c>0$. Donc $\mathbb{Z}/c\mathbb{Z}$
    est un sous anneau de $A$.
    Donc $A^\times$ contient $(\mathhb{Z}/c\mathbb{Z})^\times$.


    Si $c=2^m p_1^{r_1}...p_k^{r_k},p_i$ premier impair, alors $A^\times$ contient un sous-groupe isomorphe à $$C_{2^{m-1}}\times C_{p_1^{r_1-1}(p_1-1)}\times..\times C_{p_k^{r_k-1}(p_k-1)}$$

    Remarquons que si $k\geq 2$, alors ce groupe contient un sous-groupe isomorphe au groupe de Klein, donc n'est pas cyclique. Ainsi, $k=0,1$. Si $m\geq 3$, ce groupe n'est pas cyclique non plus.

    Si $m=2$, on a n\'{e}cessairement $k=0$, sinon on a encore un sous-groupe de Klein.

    On a donc $c:=car(A)=2,p^r, 2p^r,4, p$ premier impair.
    Dans tous les cas, $\mathbb{Z}/c\mathbb{Z}[e]$ est un quotient de $\mathbb{Z}/c\mathbb{Z}[X]/(X^n-1)$

    Il faudrait donc à étudier les unités de l'anneau $\mathbb{Z}/c\mathbb{Z}[X]/(X^n-1)$ (beuh!) , ou trouver une autre astuce.


    En tout cas, en prenant $A=\mathbb{Z}/c\mathbb{Z}$, cela montre que l'on peut avoir $n=p^{r-1}(p-1)$.

    Remarquons aussi que l'on peut avoir $n=p^r-1$ en prenant le corps fini à $p^r$ éléments.
Connectez-vous ou Inscrivez-vous pour répondre.